What are your chances of acceptance?

Calculate for all schools, your chance of acceptance.

Duke University

Your chancing factors

Extracurriculars.

student sample ap lang synthesis essay

How to Write the AP Lang Synthesis Essay + Example

Do you know how to improve your profile for college applications.

See how your profile ranks among thousands of other students using CollegeVine. Calculate your chances at your dream schools and learn what areas you need to improve right now — it only takes 3 minutes and it's 100% free.

Show me what areas I need to improve

What’s Covered:

What is the ap lang synthesis essay, how will ap scores affect my college chances.

AP English Language and Composition, commonly known as AP Lang, is one of the most engaging and popular AP classes offered at most high schools, with over 535,000 students taking the class . AP Lang tests your ability to analyze written pieces, synthesize information, write rhetorical essays, and create cohesive and concrete arguments. However, the class is rather challenging as only 62% of students were able to score a three or higher on the exam. 

The AP Lang exam has two sections. The first consists of 45 multiple choice questions which need to be completed in an hour. This portion counts for around 45% of your total score. These questions ask students to analyze written pieces and answer questions related to each respective passage.  All possible answer choices can be found within the text, and no prior knowledge of literature is needed to understand the passages.

The second section contains three free-response questions to be finished in under two hours and 15 minutes. This section counts for 55% of your score and includes the synthesis essay, the rhetorical essay, and the argumentative essay.

  • The synthesis essay requires you to read 6-7 sources and create an argument using at least three sources.
  • The rhetorical analysis essay requires you to describe how a piece of writing evokes specific meanings and symbolism.
  • The argumentative essay requires you to pick a perspective of a debate and create an argument based on the evidence provided.

In this post, we will take a look at the AP Lang synthesis essay and discuss tips and tricks to master this part of the exam. We will also provide an example of a well-written essay for review.  

The AP Lang synthesis essay is the first of three essays included in the Free Response section of the AP Lang exam. The exam presents 6-7 sources that are organized around a specific topic, with two of those sources purely visual, including a single quantitative source (like a graph or pie chart). The remaining 4-5 sources are text-based, containing around 500 words each. It’s recommended that students spend an hour on this essay—15 minute reading period, 40 minutes writing, and 5 minutes of spare time to check over work.

Each synthesis essay has a topic that all the sources will relate to. A prompt will explaining the topic and provide some background, although the topics are usually broad so you will probably know something related to the issue. It will also present a claim that students will respond to in an essay format using information from at least three of the provided sources. You will need to take a stance, either agreeing or disagreeing with the position provided in the claim. 

According to the CollegeBoard, they are looking for essays that “combine different perspectives from sources to form a support of a coherent position.” This means that you must state your claim on the topic and highlight relationships between several sources that support your specific position on the topic. Additionally, you’ll need to cite clear evidence from your sources to prove your point.

The synthesis essay counts for six points on the AP Lang exam. Students can receive 0-1 points for writing a thesis statement, 0-4 based on the incorporation of evidence and commentary, and 0-1 points based on the sophistication of thought and demonstration of complex understanding.

While this essay seems extremely overwhelming, considering there are a total of three free-response essays to complete, with proper time management and practiced skills, this essay is manageable and straightforward. In order to enhance the time management aspect of the test to the best of your ability, it is essential to divide the essay up into five key steps.

Step 1: Analyze the Prompt

As soon as the clock starts, carefully read and analyze what the prompt asks from you. It might be helpful to markup the text to identify the most critical details. You should only spend around 2 minutes reading the prompt so you have enough time to read all the sources and figure out your argument. Don’t feel like you need to immediately pick your stance on the claim right after reading the prompt. You should read the sources before you commit to your argument.

Step 2: Read the Sources Carefully

Although you are only required to use 3 of the 6-7 sources provides, make sure you read ALL of the sources. This will allow you to better understand the topic and make the most educated decision of which sources to use in your essay. Since there are a lot of sources to get through, you will need to read quickly and carefully.

Annotating will be your best friend during the reading period. Highlight and mark important concepts or lines from each passage that would be helpful in your essay. Your argument will probably begin forming in your head as you go through the passages, so you will save yourself a lot of time later on if you take a few seconds to write down notes in the margins. After you’ve finished reading a source, reflect on whether the source defends, challenges, or qualifies your argument.

You will have around 13 minutes to read through all the sources, but it’s very possible you will finish earlier if you are a fast reader. Take the leftover time to start developing your thesis and organizing your thoughts into an outline so you have more time to write. 

Step 3: Write a Strong Thesis Statement 

In order to write a good thesis statement, all you have to do is decide your stance on the claim provided in the prompt and give an overview of your evidence. You essentially have three choices on how to frame your thesis statement: You can defend, challenge or qualify a claim that’s been provided by the prompt. 

  • If you are defending the claim, your job will be to prove that the claim is correct .
  • If you are challenging the claim, your job will be to prove that the claim is incorrect .
  • If you choose to qualify the claim, your job will be to agree to a part of the claim and disagree with another part of the claim. 

A strong thesis statement will clearly state your stance without summarizing the issue or regurgitating the claim. The CollegeBoard is looking for a thesis statement that “states a defensible position and establishes a line of reasoning on the issue provided in the prompt.”

Step 4: Create a Minimal Essay Outline

Developing an outline might seem like a waste of time when you are up against the clock, but believe us, taking 5-10 minutes to outline your essay will be much more useful in the long run than jumping right into the essay.

Your outline should include your thesis statement and three main pieces of evidence that will constitute each body paragraph. Under each piece of evidence should be 2-3 details from the sources that you will use to back up your claim and some commentary on how that evidence proves your thesis.

Step 5: Write your Essay

Use the remaining 30-35 minutes to write your essay. This should be relatively easy if you took the time to mark up the sources and have a detailed outline.  Remember to add special consideration and emphasis to the commentary sections of the supporting arguments outlined in your thesis. These sentences are critical to the overall flow of the essay and where you will be explaining how the evidence supports or undermines the claim in the prompt.

Also, when referencing your sources, write the in-text citations as follows: “Source 1,” “Source 2,” “Source 3,” etc. Make sure to pay attention to which source is which in order to not incorrectly cite your sources. In-text citations will impact your score on the essay and are an integral part of the process.

After you finish writing, read through your essay for any grammatical errors or mistakes before you move onto the next essay.

Here are six must-have tips and tricks to get a good score on the synthesis essay:

  • Cite at least four sources , even though the minimum requirement is three. Remember not to plagiarize and cite everything you use in your arguments.
  • Make sure to develop a solid and clear thesis . Develop a stable stance for the claim and stick with it throughout the entire paper.
  • Don’t summarize the sources. The summary of the sources does not count as an argument. 
  • You don’t necessarily have to agree with the sources in order to cite them. Using a source to support a counterargument is still a good use of a source.
  • Cite the sources that you understand entirely . If you don’t, it could come back to bite you in the end. 
  • Use small quotes , do not quote entire paragraphs. Make sure the quote does not disrupt the flow or grammar of the sentence you write. 

student sample ap lang synthesis essay

Discover your chances at hundreds of schools

Our free chancing engine takes into account your history, background, test scores, and extracurricular activities to show you your real chances of admission—and how to improve them.

Here is an example prompt and essay from 2019 that received 5 of the 6 total points available:

In response to our society’s increasing demand for energy, large-scale wind power has drawn attention from governments and consumers as a potential alternative to traditional materials that fuel our power grids, such as coal, oil, natural gas, water, or even newer sources such as nuclear or solar power. Yet the establishment of large-scale, commercial-grade wind farms is often the subject of controversy for a variety of reasons.

Carefully read the six sources, found on the AP English Language and Composition 2019 Exam (Question 1), including the introductory information for each source. Write an essay that synthesizes material from at least three of the sources and develops your position on the most important factors that an individual or agency should consider when deciding whether to establish a wind farm.

Source A (photo)

Source B (Layton)

Source C (Seltenrich)

Source D (Brown)

Source E (Rule)

Source F (Molla)

In your response you should do the following:

  • Respond to the prompt with a thesis presents a defensible position.
  • Select and use evidence from at least 3 of the provided sources to support your line of reasoning. Indicate clearly the sources used through direct quotation, paraphrase, or summary. Sources may be cited as Source A, Source B, etc., or by using the description in parentheses.
  • Explain how the evidence supports your line of reasoning.
  • Use appropriate grammar and punctuation in communicating your argument.

[1] The situation has been known for years, and still very little is being done: alternative power is the only way to reliably power the changing world. The draw of power coming from industry and private life is overwhelming current sources of non-renewable power, and with dwindling supplies of fossil fuels, it is merely a matter of time before coal and gas fuel plants are no longer in operation. So one viable alternative is wind power. But as with all things, there are pros and cons. The main factors for power companies to consider when building wind farms are environmental boon, aesthetic, and economic factors.

[2] The environmental benefits of using wind power are well-known and proven. Wind power is, as qualified by Source B, undeniably clean and renewable. From their production requiring very little in the way of dangerous materials to their lack of fuel, besides that which occurs naturally, wind power is by far one of the least environmentally impactful sources of power available. In addition, wind power by way of gearbox and advanced blade materials, has the highest percentage of energy retention. According to Source F, wind power retains 1,164% of the energy put into the system – meaning that it increases the energy converted from fuel (wind) to electricity 10 times! No other method of electricity production is even half that efficient. The efficiency and clean nature of wind power are important to consider, especially because they contribute back to power companies economically.

[3] Economically, wind power is both a boon and a bone to electric companies and other users. For consumers, wind power is very cheap, leading to lower bills than from any other source. Consumers also get an indirect reimbursement by way of taxes (Source D). In one Texan town, McCamey, tax revenue increased 30% from a wind farm being erected in the town. This helps to finance improvements to the town. But, there is no doubt that wind power is also hurting the power companies. Although, as renewable power goes, wind is incredibly cheap, it is still significantly more expensive than fossil fuels. So, while it is helping to cut down on emissions, it costs electric companies more than traditional fossil fuel plants. While the general economic trend is positive, there are some setbacks which must be overcome before wind power can take over as truly more effective than fossil fuels.

[4] Aesthetics may be the greatest setback for power companies. Although there may be significant economic and environmental benefit to wind power, people will always fight to preserve pure, unspoiled land. Unfortunately, not much can be done to improve the visual aesthetics of the turbines. White paint is the most common choice because it “[is] associated with cleanliness.” (Source E). But, this can make it stand out like a sore thumb, and make the gargantuan machines seem more out of place. The site can also not be altered because it affects generating capacity. Sound is almost worse of a concern because it interrupts personal productivity by interrupting people’s sleep patterns. One thing for power companies to consider is working with turbine manufacturing to make the machines less aesthetically impactful, so as to garner greater public support.

[5] As with most things, wind power has no easy answer. It is the responsibility of the companies building them to weigh the benefits and the consequences. But, by balancing economics, efficiency, and aesthetics, power companies can create a solution which balances human impact with environmental preservation.

More examples can be found here at College Board.

While AP Scores help to boost your weighted GPA, or give you the option to get college credit, AP Scores don’t have a strong effect on your admissions chances . However, colleges can still see your self-reported scores, so you might not want to automatically send scores to colleges if they are lower than a 3. That being said, admissions officers care far more about your grade in an AP class than your score on the exam.

Related CollegeVine Blog Posts

student sample ap lang synthesis essay

PrepScholar

Choose Your Test

  • Search Blogs By Category
  • College Admissions
  • AP and IB Exams
  • GPA and Coursework

How to Write a Perfect Synthesis Essay for the AP Language Exam

author image

Advanced Placement (AP)

body-pencil-sharpen-notebook-1

If you're planning to take the AP Language (or AP Lang) exam , you might already know that 55% of your overall exam score will be based on three essays. The first of the three essays you'll have to write on the AP Language exam is called the "synthesis essay." If you want to earn full points on this portion of the AP Lang Exam, you need to know what a synthesis essay is and what skills are assessed by the AP Lang synthesis essay.

In this article, we'll explain the different aspects of the AP Lang synthesis essay, including what skills you need to demonstrate in your synthesis essay response in order to achieve a good score. We'll also give you a full breakdown of a real AP Lang Synthesis Essay prompt, provide an analysis of an AP Lang synthesis essay example, and give you four tips for how to write a synthesis essay.

Let's get started by taking a closer look at how the AP Lang synthesis essay works!

Synthesis Essay AP Lang: What It Is and How It Works

The AP Lang synthesis essay is the first of three essays included in the Free Response section of the AP Lang exam.

The AP Lang synthesis essay portion of the Free Response section lasts for one hour total . This hour consists of a recommended 15 minute reading period and a 40 minute writing period. Keep in mind that these time allotments are merely recommendations, and that exam takers can parse out the allotted 60 minutes to complete the synthesis essay however they choose.

Now, here's what the structure of the AP Lang synthesis essay looks like. The exam presents six to seven sources that are organized around a specific topic (like alternative energy or eminent domain, which are both past synthesis exam topics).

Of these six to seven sources, at least two are visual , including at least one quantitative source (like a graph or pie chart, for example). The remaining four to five sources are print text-based, and each one contains approximately 500 words.

In addition to six to seven sources, the AP Lang exam provides a written prompt that consists of three paragraphs. The prompt will briefly explain the essay topic, then present a claim that students will respond to in an essay that synthesizes material from at least three of the sources provided.

Here's an example prompt provided by the College Board:

Directions : The following prompt is based on the accompanying six sources.

This question requires you to integrate a variety of sources into a coherent, well-written essay. Refer to the sources to support your position; avoid mere paraphrase or summary. Your argument should be central; the sources should support this argument .

Remember to attribute both direct and indirect citations.

Introduction

Television has been influential in United States presidential elections since the 1960's. But just what is this influence, and how has it affected who is elected? Has it made elections fairer and more accessible, or has it moved candidates from pursuing issues to pursuing image?

Read the following sources (including any introductory information) carefully. Then, in an essay that synthesizes at least three of the sources for support, take a position that defends, challenges, or qualifies the claim that television has had a positive impact on presidential elections.

Refer to the sources as Source A, Source B, etc.; titles are included for your convenience.

Source A (Campbell) Source B (Hart and Triece) Source C (Menand) Source D (Chart) Source E (Ranney) Source F (Koppel)

Like we mentioned earlier, this prompt gives you a topic — which it briefly explains — then asks you to take a position. In this case, you'll have to choose a stance on whether television has positively or negatively affected U.S. elections. You're also given six sources to evaluate and use in your response. Now that you have everything you need, now your job is to write an amazing synthesis essay.

But what does "synthesize" mean, exactly? According to the CollegeBoard, when an essay prompt asks you to synthesize, it means that you should "combine different perspectives from sources to form a support of a coherent position" in writing. In other words, a synthesis essay asks you to state your claim on a topic, then highlight the relationships between several sources that support your claim on that topic. Additionally, you'll need to cite specific evidence from your sources to prove your point.

The synthesis essay counts for six of the total points on the AP Lang exam . Students can receive 0-1 points for writing a thesis statement in the essay, 0-4 based on incorporation of evidence and commentary, and 0-1 points based on sophistication of thought and demonstrated complex understanding of the topic.

You'll be evaluated based on how effectively you do the following in your AP Lang synthesis essay:

Write a thesis that responds to the exam prompt with a defensible position

Provide specific evidence that to support all claims in your line of reasoning from at least three of the sources provided, and clearly and consistently explain how the evidence you include supports your line of reasoning

Demonstrate sophistication of thought by either crafting a thoughtful argument, situating the argument in a broader context, explaining the limitations of an argument

Make rhetorical choices that strengthen your argument and/or employ a vivid and persuasive style throughout your essay.

If your synthesis essay meets the criteria above, then there's a good chance you'll score well on this portion of the AP Lang exam!

If you're looking for even more information on scoring, the College Board has posted the AP Lang Free Response grading rubric on its website. ( You can find it here. ) We recommend taking a close look at it since it includes additional details about the synthesis essay scoring.

body-chisel-break-apart

Don't be intimidated...we're going to teach you how to break down even the hardest AP synthesis essay prompt.

Full Breakdown of a Real AP Lang Synthesis Essay Prompt

In this section, we'll teach you how to analyze and respond to a synthesis essay prompt in five easy steps, including suggested time frames for each step of the process.

Step 1: Analyze the Prompt

The very first thing to do when the clock starts running is read and analyze the prompt. To demonstrate how to do this, we'll look at the sample AP Lang synthesis essay prompt below. This prompt comes straight from the 2018 AP Lang exam:

Eminent domain is the power governments have to acquire property from private owners for public use. The rationale behind eminent domain is that governments have greater legal authority over lands within their dominion than do private owners. Eminent domain has been instituted in one way or another throughout the world for hundreds of years.

Carefully read the following six sources, including the introductory information for each source. Then synthesize material from at least three of the sources and incorporate it into a coherent, well-developed essay that defends, challenges, or qualifies the notion that eminent domain is productive and beneficial.

Your argument should be the focus of your essay. Use the sources to develop your argument and explain the reasoning for it. Avoid merely summarizing the sources. Indicate clearly which sources you are drawing from, whether through direct quotation, paraphrase, or summary. You may cite the sources as Source A, Source B, etc., or by using the descriptions in parentheses.

On first read, you might be nervous about how to answer this prompt...especially if you don't know what eminent domain is! But if you break the prompt down into chunks, you'll be able to figure out what the prompt is asking you to do in no time flat.

To get a full understanding of what this prompt wants you to do, you need to identify the most important details in this prompt, paragraph by paragraph. Here's what each paragraph is asking you to do:

  • Paragraph 1: The prompt presents and briefly explains the topic that you'll be writing your synthesis essay about. That topic is the concept of eminent domain.
  • Paragraph 2: The prompt presents a specific claim about the concept of eminent domain in this paragraph: Eminent domain is productive and beneficial. This paragraph instructs you to decide whether you want to defend, challenge, or qualify that claim in your synthesis essay , and use material from at least three of the sources provided in order to do so.
  • Paragraph 3: In the last paragraph of the prompt, the exam gives you clear instructions about how to approach writing your synthesis essay . First, make your argument the focus of the essay. Second, use material from at least three of the sources to develop and explain your argument. Third, provide commentary on the material you include, and provide proper citations when you incorporate quotations, paraphrases, or summaries from the sources provided.

So basically, you'll have to agree with, disagree with, or qualify the claim stated in the prompt, then use at least three sources substantiate your answer. Since you probably don't know much about eminent domain, you'll probably decide on your position after you read the provided sources.

To make good use of your time on the exam, you should spend around 2 minutes reading the prompt and making note of what it's asking you to do. That will leave you plenty of time to read the sources provided, which is the next step to writing a synthesis essay.

Step 2: Read the Sources Carefully

After you closely read the prompt and make note of the most important details, you need to read all of the sources provided. It's tempting to skip one or two sources to save time--but we recommend you don't do this. That's because you'll need a thorough understanding of the topic before you can accurately address the prompt!

For the sample exam prompt included above, there are six sources provided. We're not going to include all of the sources in this article, but you can view the six sources from this question on the 2018 AP Lang exam here . The sources include five print-text sources and one visual source, which is a cartoon.

As you read the sources, it's important to read quickly and carefully. Don't rush! Keep your pencil in hand to quickly mark important passages that you might want to use as evidence in your synthesis. While you're reading the sources and marking passages, you want to think about how the information you're reading influences your stance on the issue (in this case, eminent domain).

When you finish reading, take a few seconds to summarize, in a phrase or sentence, whether the source defends, challenges, or qualifies whether eminent domain is beneficial (which is the claim in the prompt) . Though it might not feel like you have time for this, it's important to give yourself these notes about each source so you know how you can use each one as evidence in your essay.

Here's what we mean: say you want to challenge the idea that eminent domain is useful. If you've jotted down notes about each source and what it's saying, it will be easier for you to pull the relevant information into your outline and your essay.

So how much time should you spend reading the provided sources? The AP Lang exam recommends taking 15 minutes to read the sources . If you spend around two of those minutes reading and breaking down the essay prompt, it makes sense to spend the remaining 13 minutes reading and annotating the sources.

If you finish reading and annotating early, you can always move on to drafting your synthesis essay. But make sure you're taking your time and reading carefully! It's better to use a little extra time reading and understanding the sources now so that you don't have to go back and re-read the sources later.

body-weightlifting-lift-strong

A strong thesis will do a lot of heavy lifting in your essay. (See what we did there?)

Step 3: Write a Strong Thesis Statement

After you've analyzed the prompt and thoroughly read the sources, the next thing you need to do in order to write a good synthesis essay is write a strong thesis statement .

The great news about writing a thesis statement for this synthesis essay is that you have all the tools you need to do it at your fingertips. All you have to do in order to write your thesis statement is decide what your stance is in relationship to the topic provided.

In the example prompt provided earlier, you're essentially given three choices for how to frame your thesis statement: you can either defend, challenge, or qualify a claim that's been provided by the prompt, that eminent domain is productive and beneficial . Here's what that means for each option:

If you choose to defend the claim, your job will be to prove that the claim is correct . In this case, you'll have to show that eminent domain is a good thing.

If you choose to challenge the claim, you'll argue that the claim is incorrect. In other words, you'll argue that eminent domain isn't productive or beneficial.

If you choose to qualify, that means you'll agree with part of the claim, but disagree with another part of the claim. For instance, you may argue that eminent domain can be a productive tool for governments, but it's not beneficial for property owners. Or maybe you argue that eminent domain is useful in certain circumstances, but not in others.

When you decide whether you want your synthesis essay to defend, challenge, or qualify that claim, you need to convey that stance clearly in your thesis statement. You want to avoid simply restating the claim provided in the prompt, summarizing the issue without making a coherent claim, or writing a thesis that doesn't respond to the prompt.

Here's an example of a thesis statement that received full points on the eminent domain synthesis essay:

Although eminent domain can be misused to benefit private interests at the expense of citizens, it is a vital tool of any government that intends to have any influence on the land it governs beyond that of written law.

This thesis statement received full points because it states a defensible position and establishes a line of reasoning on the issue of eminent domain. It states the author's position (that some parts of eminent domain are good, but others are bad), then goes on to explain why the author thinks that (it's good because it allows the government to do its job, but it's bad because the government can misuse its power.)

Because this example thesis statement states a defensible position and establishes a line of reasoning, it can be elaborated upon in the body of the essay through sub-claims, supporting evidence, and commentary. And a solid argument is key to getting a six on your synthesis essay for AP Lang!

Looking for help studying for your AP exam? Our one-on-one online AP tutoring services can help you prepare for your AP exams. Get matched with a top tutor who got a high score on the exam you're studying for!

Step 4: Create a Bare-Bones Essay Outline

Once you've got your thesis statement drafted, you have the foundation you need to develop a bare bones outline for your synthesis essay. Developing an outline might seem like it's a waste of your precious time, but if you develop your outline well, it will actually save you time when you start writing your essay.

With that in mind, we recommend spending 5 to 10 minutes outlining your synthesis essay . If you use a bare-bones outline like the one below, labeling each piece of content that you need to include in your essay draft, you should be able to develop out the most important pieces of the synthesis before you even draft the actual essay.

To help you see how this can work on test day, we've created a sample outline for you. You can even memorize this outline to help you out on test day! In the outline below, you'll find places to fill in a thesis statement, body paragraph topic sentences, evidence from the sources provided, and commentary :

  • Present the context surrounding the essay topic in a couple of sentences (this is a good place to use what you learned about the major opinions or controversies about the topic from reading your sources).
  • Write a straightforward, clear, and concise thesis statement that presents your stance on the topic
  • Topic sentence presenting first supporting point or claim
  • Evidence #1
  • Commentary on Evidence #1
  • Evidence #2 (if needed)
  • Commentary on Evidence #2 (if needed)
  • Topic sentence presenting second supporting point or claim
  • Topic sentence presenting three supporting point or claim
  • Sums up the main line of reasoning that you developed and defended throughout the essay
  • Reiterates the thesis statement

Taking the time to develop these crucial pieces of the synthesis in a bare-bones outline will give you a map for your final essay. Once you have a map, writing the essay will be much easier.

Step 5: Draft Your Essay Response

The great thing about taking a few minutes to develop an outline is that you can develop it out into your essay draft. After you take about 5 to 10 minutes to outline your synthesis essay, you can use the remaining 30 to 35 minutes to draft your essay and review it.

Since you'll outline your essay before you start drafting, writing the essay should be pretty straightforward. You'll already know how many paragraphs you're going to write, what the topic of each paragraph will be, and what quotations, paraphrases, or summaries you're going to include in each paragraph from the sources provided. You'll just have to fill in one of the most important parts of your synthesis—your commentary.

Commentaries are your explanation of why your evidence supports the argument you've outlined in your thesis. Your commentary is where you actually make your argument, which is why it's such a critical part of your synthesis essay.

When thinking about what to say in your commentary, remember one thing the AP Lang synthesis essay prompt specifies: don't just summarize the sources. Instead, as you provide commentary on the evidence you incorporate, you need to explain how that evidence supports or undermines your thesis statement . You should include commentary that offers a thoughtful or novel perspective on the evidence from your sources to develop your argument.

One very important thing to remember as you draft out your essay is to cite your sources. The AP Lang exam synthesis essay prompt indicates that you can use generic labels for the sources provided (e.g. "Source 1," "Source 2," "Source 3," etc.). The exam prompt will indicate which label corresponds with which source, so you'll need to make sure you pay attention and cite sources accurately. You can cite your sources in the sentence where you introduce a quote, summary, or paraphrase, or you can use a parenthetical citation. Citing your sources affects your score on the synthesis essay, so remembering to do this is important.

body-green-arrow-down

Keep reading for a real-life example of a great AP synthesis essay response!

Real-Life AP Synthesis Essay Example and Analysis

If you're still wondering how to write a synthesis essay, examples of real essays from past AP Lang exams can make things clearer. These real-life student AP synthesis essay responses can be great for helping you understand how to write a synthesis essay that will knock the graders' socks off .

While there are multiple essay examples online, we've chosen one to take a closer look at. We're going to give you a brief analysis of one of these example student synthesis essays from the 2019 AP Lang Exam below!

Example Synthesis Essay AP Lang Response

To get started, let's look at the official prompt for the 2019 synthesis essay:

In response to our society's increasing demand for energy, large-scale wind power has drawn attention from governments and consumers as a potential alternative to traditional materials that fuel our power grids, such as coal, oil, natural gas, water, or even newer sources such as nuclear or solar power. Yet the establishment of large-scale, commercial-grade wind farms is often the subject of controversy for a variety of reasons.

Carefully read the six sources, found on the AP English Language and Composition 2019 Exam (Question 1), including the introductory information for each source. Write an essay that synthesizes material from at least three of the sources and develops your position on the most important factors that an individual or agency should consider when deciding whether to establish a wind farm.

Source A (photo) Source B (Layton) Source C (Seltenrich) Source D (Brown) Source E (Rule) Source F (Molla)

In your response you should do the following:

  • Respond to the prompt with a thesis presents a defensible position.
  • Select and use evidence from at least 3 of the provided sources to support your line of reasoning. Indicate clearly the sources used through direct quotation, paraphrase, or summary. Sources may be cited as Source A, Source B, etc., or by using the description in parentheses.
  • Explain how the evidence supports your line of reasoning.
  • Use appropriate grammar and punctuation in communicating your argument.

Now that you know exactly what the prompt asked students to do on the 2019 AP Lang synthesis essay, here's an AP Lang synthesis essay example, written by a real student on the AP Lang exam in 2019:

[1] The situation has been known for years, and still very little is being done: alternative power is the only way to reliably power the changing world. The draw of power coming from industry and private life is overwhelming current sources of non-renewable power, and with dwindling supplies of fossil fuels, it is merely a matter of time before coal and gas fuel plants are no longer in operation. So one viable alternative is wind power. But as with all things, there are pros and cons. The main factors for power companies to consider when building wind farms are environmental boon, aesthetic, and economic factors.

[2] The environmental benefits of using wind power are well-known and proven. Wind power is, as qualified by Source B, undeniably clean and renewable. From their production requiring very little in the way of dangerous materials to their lack of fuel, besides that which occurs naturally, wind power is by far one of the least environmentally impactful sources of power available. In addition, wind power by way of gearbox and advanced blade materials, has the highest percentage of energy retention. According to Source F, wind power retains 1,164% of the energy put into the system – meaning that it increases the energy converted from fuel (wind) to electricity 10 times! No other method of electricity production is even half that efficient. The efficiency and clean nature of wind power are important to consider, especially because they contribute back to power companies economically.

[3] Economically, wind power is both a boon and a bone to electric companies and other users. For consumers, wind power is very cheap, leading to lower bills than from any other source. Consumers also get an indirect reimbursement by way of taxes (Source D). In one Texan town, McCamey, tax revenue increased 30% from a wind farm being erected in the town. This helps to finance improvements to the town. But, there is no doubt that wind power is also hurting the power companies. Although, as renewable power goes, wind is incredibly cheap, it is still significantly more expensive than fossil fuels. So, while it is helping to cut down on emissions, it costs electric companies more than traditional fossil fuel plants. While the general economic trend is positive, there are some setbacks which must be overcome before wind power can take over as truly more effective than fossil fuels.

[4] Aesthetics may be the greatest setback for power companies. Although there may be significant economic and environmental benefit to wind power, people will always fight to preserve pure, unspoiled land. Unfortunately, not much can be done to improve the visual aesthetics of the turbines. White paint is the most common choice because it "[is] associated with cleanliness." (Source E). But, this can make it stand out like a sore thumb, and make the gargantuan machines seem more out of place. The site can also not be altered because it affects generating capacity. Sound is almost worse of a concern because it interrupts personal productivity by interrupting people's sleep patterns. One thing for power companies to consider is working with turbine manufacturing to make the machines less aesthetically impactful, so as to garner greater public support.

[5] As with most things, wind power has no easy answer. It is the responsibility of the companies building them to weigh the benefits and the consequences. But, by balancing economics, efficiency, and aesthetics, power companies can create a solution which balances human impact with environmental preservation.

And that's an entire AP Lang synthesis essay example, written in response to a real AP Lang exam prompt! It's important to remember AP Lang exam synthesis essay prompts are always similarly structured and worded, and students often respond in around the same number of paragraphs as what you see in the example essay response above.

Next, let's analyze this example essay and talk about what it does effectively, where it could be improved upon, and what score past exam scorers awarded it.

To get started on an analysis of the sample synthesis essay, let's look at the scoring commentary provided by the College Board:

  • For development of thesis, the essay received 1 out of 1 possible points
  • For evidence and commentary, the essay received 4 out of 4 possible points
  • For sophistication of thought, the essay received 0 out of 1 possible points.

This means that the final score for this example essay was a 5 out of 6 possible points . Let's look more closely at the content of the example essay to figure out why it received this score breakdown.

Thesis Development

The thesis statement is one of the three main categories that is taken into consideration when you're awarded points on this portion of the exam. This sample essay received 1 out of 1 total points.

Now, here's why: the thesis statement clearly and concisely conveys a position on the topic presented in the prompt--alternative energy and wind power--and defines the most important factors that power companies should consider when deciding whether to establish a wind farm.

Evidence and Commentary

The second key category taken into consideration when synthesis exams are evaluated is incorporation of evidence and commentary. This sample received 4 out of 4 possible points for this portion of the synthesis essay. At bare minimum, this sample essay meets the requirement mentioned in the prompt that the writer incorporate evidence from at least three of the sources provided.

On top of that, the writer does a good job of connecting the incorporated evidence back to the claim made in the thesis statement through effective commentary. The commentary in this sample essay is effective because it goes beyond just summarizing what the provided sources say. Instead, it explains and analyzes the evidence presented in the selected sources and connects them back to supporting points the writer makes in each body paragraph.

Finally, the writer of the essay also received points for evidence and commentary because the writer developed and supported a consistent line of reasoning throughout the essay . This line of reasoning is summed up in the fourth paragraph in the following sentence: "One thing for power companies to consider is working with turbine manufacturing to make the machines less aesthetically impactful, so as to garner greater public support."

Because the writer did a good job consistently developing their argument and incorporating evidence, they received full marks in this category. So far, so good!

Sophistication of Thought

Now, we know that this essay received a score of 5 out of 6 total points, and the place where the writer lost a point was on the basis of sophistication of thought, for which the writer received 0 out of 1 points. That's because this sample essay makes several generalizations and vague claims where it could have instead made specific claims that support a more balanced argument.

For example, in the following sentence from the 5th paragraph of the sample essay, the writer misses the opportunity to state specific possibilities that power companies should consider for wind energy . Instead, the writer is ambiguous and non-committal, saying, "As with most things, wind power has no easy answer. It is the responsibility of the companies building them to weigh the benefits and consequences."

If the writer of this essay was interested in trying to get that 6th point on the synthesis essay response, they could consider making more specific claims. For instance, they could state the specific benefits and consequences power companies should consider when deciding whether to establish a wind farm. These could include things like environmental impacts, economic impacts, or even population density!

Despite losing one point in the last category, this example synthesis essay is a strong one. It's well-developed, thoughtfully written, and advances an argument on the exam topic using evidence and support throughout.

body-number-four-post-it-note

4 Tips for How to Write a Synthesis Essay

AP Lang is a timed exam, so you have to pick and choose what you want to focus on in the limited time you're given to write the synthesis essay. Keep reading to get our expert advice on what you should focus on during your exam.

Tip 1: Read the Prompt First

It may sound obvious, but when you're pressed for time, it's easy to get flustered. Just remember: when it comes time to write the synthesis essay, read the prompt first !

Why is it so important to read the prompt before you read the sources? Because when you're aware of what kind of question you're trying to answer, you'll be able to read the sources more strategically. The prompt will help give you a sense of what claims, points, facts, or opinions to be looking for as you read the sources.

Reading the sources without having read the prompt first is kind of like trying to drive while wearing a blindfold: you can probably do it, but it's likely not going to end well!

Tip 2: Make Notes While You Read

During the 15-minute reading period at the beginning of the synthesis essay, you'll be reading through the sources as quickly as you can. After all, you're probably anxious to start writing!

While it's definitely important to make good use of your time, it's also important to read closely enough that you understand your sources. Careful reading will allow you to identify parts of the sources that will help you support your thesis statement in your essay, too.

As you read the sources, consider marking helpful passages with a star or check mark in the margins of the exam so you know which parts of the text to quickly re-read as you form your synthesis essay. You might also consider summing up the key points or position of each source in a sentence or a few words when you finish reading each source during the reading period. Doing so will help you know where each source stands on the topic given and help you pick the three (or more!) that will bolster your synthesis argument.

Tip 3: Start With the Thesis Statement

If you don't start your synthesis essay with a strong thesis statement, it's going to be tough to write an effective synthesis essay. As soon as you finish reading and annotating the provided sources, the thing you want to do next is write a strong thesis statement.

According to the CollegeBoard grading guidelines for the AP Lang synthesis essay, a strong thesis statement will respond to the prompt— not restate or rephrase the prompt. A good thesis will take a clear, defensible position on the topic presented in the prompt and the sources.

In other words, to write a solid thesis statement to guide the rest of your synthesis essay, you need to think about your position on the topic at hand and then make a claim about the topic based on your position. This position will either be defending, challenging, or qualifying the claim made in the essay's prompt.

The defensible position that you establish in your thesis statement will guide your argument in the rest of the essay, so it's important to do this first. Once you have a strong thesis statement, you can begin outlining your essay.

Tip 4: Focus on Your Commentary

Writing thoughtful, original commentary that explains your argument and your sources is important. In fact, doing this well will earn you four points (out of a total of six)!

AP Lang provides six to seven sources for you on the exam, and you'll be expected to incorporate quotations, paraphrases, or summaries from at least three of those sources into your synthesis essay and interpret that evidence for the reader.

While incorporating evidence is very important, in order to get the extra point for "sophistication of thought" on the synthesis essay, it's important to spend more time thinking about your commentary on the evidence you choose to incorporate. The commentary is your chance to show original thinking, strong rhetorical skills, and clearly explain how the evidence you've included supports the stance you laid out in your thesis statement.

To earn the 6th possible point on the synthesis essay, make sure your commentary demonstrates a nuanced understanding of the source material, explains this nuanced understanding, and places the evidence incorporated from the sources in conversation with each other. To do this, make sure you're avoiding vague language. Be specific when you can, and always tie your commentary back to your thesis!

body-person-arrows-next

What's Next?

There's a lot more to the AP Language exam than just the synthesis essay. Be sure to check out our expert guide to the entire exam , then learn more about the tricky multiple choice section .

Is the AP Lang exam hard...or is it easy? See how it stacks up to other AP tests on our list of the hardest AP exams .

Did you know there are technically two English AP exams? You can learn more about the second English AP test, the AP Literature exam, in this article . And if you're confused about whether you should take the AP Lang or AP Lit test , we can help you make that decision, too.

Want to improve your SAT score by 160 points or your ACT score by 4 points?   We've written a guide for each test about the top 5 strategies you must be using to have a shot at improving your score. Download them for free now:

Trending Now

How to Get Into Harvard and the Ivy League

How to Get a Perfect 4.0 GPA

How to Write an Amazing College Essay

What Exactly Are Colleges Looking For?

ACT vs. SAT: Which Test Should You Take?

When should you take the SAT or ACT?

Get Your Free

PrepScholar

Find Your Target SAT Score

Free Complete Official SAT Practice Tests

How to Get a Perfect SAT Score, by an Expert Full Scorer

Score 800 on SAT Math

Score 800 on SAT Reading and Writing

How to Improve Your Low SAT Score

Score 600 on SAT Math

Score 600 on SAT Reading and Writing

Find Your Target ACT Score

Complete Official Free ACT Practice Tests

How to Get a Perfect ACT Score, by a 36 Full Scorer

Get a 36 on ACT English

Get a 36 on ACT Math

Get a 36 on ACT Reading

Get a 36 on ACT Science

How to Improve Your Low ACT Score

Get a 24 on ACT English

Get a 24 on ACT Math

Get a 24 on ACT Reading

Get a 24 on ACT Science

Stay Informed

Get the latest articles and test prep tips!

Follow us on Facebook (icon)

Ashley Sufflé Robinson has a Ph.D. in 19th Century English Literature. As a content writer for PrepScholar, Ashley is passionate about giving college-bound students the in-depth information they need to get into the school of their dreams.

Ask a Question Below

Have any questions about this article or other topics? Ask below and we'll reply!

logo-type-white

AP® English Language

How to ace the ap® english language and composition synthesis essay.

  • The Albert Team
  • Last Updated On: March 1, 2022

how_to_ace_the_ap_english english language synthesis essay

The newest section of the AP® English Language and Composition Exam, the synthesis essay, is one of three essays you will be completing during the examination’s 2-hour free-response period. However, you’ll also have a 15-minute reading and planning period just for this essay, and if you use this time to plan effectively, you can’t go wrong.

Before we get into specific advice on how to handle the AP® English Language and Composition synthesis essay, you need to know what this part of the test really is. It is very similar to the argumentative essay you will also write as part of this exam, except that you are provided with a wealth of source material from which to draw some support for your ideas.

While this in some ways makes the AP® English Language and Composition synthesis essay easier than the argument essay (because you can use quotations, point to authoritative sources for support, etc.), there is an extra element of complexity, and the AP® readers want to see how well you can sort through your source material and put it to good use – which makes planning all that much more important. This brings us to our first tip…

1. Use Your 15-Minute Planning Period Wisely.

The main purpose of this 15-minute period is to give you time to read the source materials. This essay will present you with several sources providing different information about or opinions on a certain topic. Make sure you don’t just skim them, but read them closely – make notes, underline key sections you may want to quote later, etc.

You should also begin outlining your essay and considering your opinion on the subject; have this opinion in mind before you start writing the essay, as you will use it to construct your thesis.

You’ve already learned how to structure persuasive essays in this class and in other classes you have taken; put that knowledge to good use now, and have your main points set out before you start writing. Try to have a thesis statement written by the time you start the essay – your thesis should establish your opinion and the general reasons you feel this way; the rest of your essay will go on to justify and exemplify these reasons. Also write down some of the main points upon which you will base subsequent paragraphs and mark quotes or sections of the sources you can use in each of these paragraphs.

2. Evaluate Your Sources.

ap exams score

Every source you can use for the AP® Language and Composition synthesis essay will have a small box above it explaining where it comes from and who said it – to see exactly what this looks like, check out the free synthesis essay sample questions at AP® Central. There are also public sample questions available there for the rest of the AP® English and Composition Exam .

Keep all information about your sources in mind when you’re quoting them or using them to support your arguments. What journal an article appeared in can say a great deal about its potential biases. For example, consider a question on the environmental impacts of corporate practices – an environmental journal is obviously going to be biased in favor of more environmental regulation, while a report from a company spokesperson will probably gloss over some of the negative impacts of his company. Think critically.

3. Keep Your Tone Consistent.

There is no hard-and-fast advice about what tone you should take – some students try to inject a little humor into their essays while others prefer to be as serious as possible, some are extremely critical and others more accepting. However, the one thing you really have to do while writing the AP® Language and Composition synthesis essay (or any other essay) is keep your tone consistent. Jot some tone-related ideas down as you outline during the 15-minute reading period, and keep in mind everything you’ve learned about tone and other aspects of rhetoric so far this year.

4. Use Rhetorical Technique to Your Advantage!

The various rhetorical practices you’ve been learning about all year can be put to good use here. This class and this test aren’t just about recognizing and analyzing these techniques when others use them, but about preparing you for college and your career by teaching you how to use them effectively yourself. However, this isn’t just about writing a beautiful essay, so read on to Tip # 5!

5. Your Argument Must be Well-Crafted.

The AP® English Language and Composition Exam synthesis essay does not have right or wrong answers; rather, it asks you for your opinion. The AP® Examiner cannot take points off because she disagrees with you. However, you must show logical basis for your opinion, drawing on both the sources AND your own knowledge and experience.

To do this, make sure you have a clear and complete thesis. Make sure the ideas expressed in the beginning of each paragraph or section support the thesis, and that you in turn show how those ideas are supported by a source or through your own knowledge and experience. Don’t generalize or write anything down that you can’t support.

Looking for AP® English Language practice?

Kickstart your AP® English Language prep with Albert. Start your AP® exam prep today .

Interested in a school license?​

Popular posts.

AP® Physics I score calculator

AP® Score Calculators

Simulate how different MCQ and FRQ scores translate into AP® scores

student sample ap lang synthesis essay

AP® Review Guides

The ultimate review guides for AP® subjects to help you plan and structure your prep.

student sample ap lang synthesis essay

Core Subject Review Guides

Review the most important topics in Physics and Algebra 1 .

student sample ap lang synthesis essay

SAT® Score Calculator

See how scores on each section impacts your overall SAT® score

student sample ap lang synthesis essay

ACT® Score Calculator

See how scores on each section impacts your overall ACT® score

student sample ap lang synthesis essay

Grammar Review Hub

Comprehensive review of grammar skills

student sample ap lang synthesis essay

AP® Posters

Download updated posters summarizing the main topics and structure for each AP® exam.

How to Write the AP Lang Synthesis Essay

student sample ap lang synthesis essay

AP Lang test is the logical conclusion to the introductory college English composition course. And its most important (and often difficult) part is the AP Lang synthesis essay. Despite it being the very basic layer of your future composition skills, it’s a very complicated challenge to approach unprepared. Besides, it's details may change year to year. So let’s have a look with our coursework writing services team at what your AP Lang exam 2022 might look like.

What is AP Lang?

AP Lang is a relatively lengthy test. There are several AP rubrics that a student must be well-versed in to hope to pass it. The first section includes reading and writing, while the second is slightly more freeform and includes three different types of essays.

Among those three, the most interesting and, coincidentally, oftentimes the hardest to deal with is the AP Lang synthesis essay rubric. Today will focus on it specifically to make sure you know exactly what you’re going to be facing during your test.

What Is a Synthesis Essay AP Lang?

At its core, the AP Lang synthesis essay is a pretty straightforward part of the AP Lang test. It might look pretty similar to the reading section of the exam. However, simply finding the right information isn’t enough. When writing a synthesis essay, you should not only gather the data but also distill it into your personal opinion.

This fine line may seem difficult to spot, but it is there. And it’s that small difference that can make or break your exam run. So try to follow the steps one by one and not lose focus. Writing a good synthesis essay is as easy as following the rules. If you feel this task is too difficult for you, you can leave us your ' write an essay for me ' request and we will do it for you.

AP Lang Synthesis Essay Outline

Looking through AP Lang essay examples, you might notice that the overall structure doesn’t really differ too much from your standard essay outline. You have your introduction, your body, and your conclusion. But the important thing to note is where your arguments are supposed to come from.

You’re not supposed to just go off on a rant. The task requires you to base your supporting evidence on at least three sources. And you will have to ensure your essay has solid roots. Here’s what a basic AP Lang exam synthesis essay outline should look like:

  • Introduction

Provide sufficient context for the topic you are about to cover. You can do a quick overview of prevailing opinions you have grasped while browsing through your source materials.

Write a short and compelling thesis statement. This will be your ground zero for the rest of the essay. So make sure it reflects your opinion. What is a thesis statement you can read in our special article.

  • Body Paragraphs

Dedicate at least one paragraph to every source you’re using. Start with presenting the evidence you have gathered from that source and go on to explain how it formed your opinion on the topic and why it should be considered.

Quickly go through your line of reasoning and reinforce what you have already covered. Finish up with restating your thesis as you’re supposed to logically arrive at it after all the evidence you have presented. That’s how you write a conclusion properly.

Different Forms and Types of Synthesis Essay: Explanatory vs. Argumentative Synthesis Essays

When it comes to writing a synthesis essay AP Lang, there are several types of essays you should consider. The most common ones are the AP Lang argument essay and explanatory essay. The clues as to how each of them should look are hidden within their names but let’s go over them to clear any confusion.

An explanatory essay’s goal is to go over a certain topic, discuss it in detail, and ultimately show a high level of understanding of the said topic. You don’t necessarily have to get into a heated argument with the reader trying to convince them of something. All you need to do is create an impartial overview.

On the other hand, an argumentative essay has to do with personal opinions. And while there is a time and a place for bias, it still has to be as impartial and factual as possible. When proving your point, try not to devolve into emotional arguments but stick to logic and cold truths. This will make your argument way more solid.

Synthesis Essay Structure

In the general case, you don’t really need to look for a synthesis essay AP Lang example to get a solid grasp on how its structure should look like. You can safely fall back on your high school essay writing knowledge, and you’ll be mostly safe.

What you should pay attention to is your writing style and content. A synthesis essay is identified less by its structure and more by the way you form and present your arguments to the reader. It’s when you get a specific essay type (like an argumentative essay) that you should pay attention to slight changes in format.

Argument Essay Structure

The best way to understand argumentative essay structure is to study any well-written AP Lang argument essay example. Standard AP Lang essays have very distinctive features that are very easy to spot and emulate. They follow a very rigid form and employ specific rhetorical devices that you’ll be able to pick up after you analyze them once or twice quickly.

How Many Paragraphs Should an AP Lang Synthesis Essay Be?

The number of paragraphs in an AP Lang synthesis essay can indeed make a difference. Your arguments should be concise and pointed. Spreading them out throughout many paragraphs may seem like a good idea to fill in the space. But it’s actually detrimental to your final score. You can get a basic understanding of what your score is going to be using an AP Lang score calculator.

The same goes for too few paragraphs. Don’t even try to squeeze your entire line of thought into a single body paragraph. Generally, the minimum number of sources you should address is three. Any less, and you are getting a lower score. So try to keep it somewhere in the middle. Three to five body paragraphs is an optimal number. Don’t forget to add an intro and a conclusion to it and you’re all set. A well-written essay has a clear and easily identifiable structure.

How to Write AP Lang Synthesis Essay: Guide

How to Write the AP Lang Synthesis Essay

In order to write a decent essay, all you have to do is follow these simple steps. Performing a rhetorical analysis essay example, AP Lang won’t give you insight into how it was built from the ground up. But looking at this list might.

Step 1. Read the Prompt

It may sound like a no-brainer. But it’s actually more important than you can imagine. Don’t skip right past this step. It’s very easy to misunderstand the task under stress. And if you do slip up in the beginning - the entirety of your work after that is wasted.

Step 2. Analyze the Sources Carefully

The same goes for your sources. Take your time reading them. Try to spot every smallest detail, as even a single one can help you better incorporate your evidence into the body of your essay. You can begin outlining the general points of your essay in your head at this point.

Step 3. Come Up with a Strong Thesis Statement

Your thesis statement is the baseline of your writing. Make it short and clear. Try not to overthink it too much.

Step 4. Fill in Your Essay Outline

Start filling out your outline step by step. You don’t have to go from top to bottom. If you feel like you’re struggling - skip to the next part and return to the problem paragraph later. The use of rhetorical devices AP Lang is also pretty important. So once you flesh out your essay a bit, spend some time trying to come up with the perfect wording.

Step 5. Finalize

The first finished version of your essay is a draft. Don’t be hasty to turn it in. Read over it a couple of times. Make sure everything is in order. You can switch some of the parts around or rewrite some sections if you have the time. Ideally, at this stage you should have enough time to eliminate all grammatical errors that may still be present in your essay. Polish it to perfection.

Useful Tips

Here are some useful tips that might make the writing process a bit easier for you:

  • Use either APA or Chicago style to cite your sources
  • Have a schedule to understand how much time you have for each section
  • Leave as much time as you can for editing and proofreading
  • You can never over study the source material. Spend as much time as you can reading into it
  • Don’t linger on the surface of your essay subject. Dive in and show your complex understanding of the material
  • Avoid using private life anecdotes to support your case unless the essay type specifically allows it. These don’t make for a convincing argument.
  • Use as many supporting arguments as you can but make sure they are actually solid and relevant to your thesis
  • Check with your thesis from time to time. The entirety of your text should align with it

Need help with academic deadlines?

Falling back on your deadlines? Use our term paper writing services to relieve you while you get back on your feet.

AP Lang Essay Prompts

Here are some interesting prompts. Some of them could be found in the previous iterations of the test; you may have spotted them in some of the AP Lang essay examples. Others are there to help you practice for the AP Lang exam 2022.

  • The John F. Kennedy Presidential Library and Museum, dedicated in 1979, was founded in memory of the president and contained archives pertaining to his administration. On June 24, 1985, then President Ronald Reagan joined members of the Kennedy family at a fundraising event to help the Kennedy Library Foundation create an endowment to fund and support the presidential library. The following is an excerpt from the speech Reagan gave at that event. Read the passage carefully. Write an essay that analyzes the rhetorical choices Reagan makes to achieve his purpose of paying tribute to John F. Kennedy.
  • On August 29, 2009, then-President Barack Obama delivered a eulogy at the funeral of Senator Ted Kennedy in Boston, Massachusetts. Kennedy served in the United States Senate from 1962 until his death. Obama served with him in the Senate from 2005 until Obama was elected president in 2008. The following is an excerpt from Obama’s speech. Read the passage carefully. Write an essay that analyzes the rhetorical choices Obama makes to achieve his purpose of praising and memorializing Kennedy.
  • On April 9, 1964, Claudia “Lady Bird” Johnson, who was at the time the First Lady of the United States, gave the following speech at the first-anniversary luncheon of the Eleanor Roosevelt Memorial Foundation. The foundation is a nonprofit division of the Franklin D. Roosevelt Presidential Library dedicated to the works of former First Lady Eleanor Roosevelt, who passed away in 1962. Read the passage carefully. Write an essay that analyzes the rhetorical choices Johnson makes to achieve her purpose of paying tribute to Eleanor Roosevelt.

In your response, you should do the following:

• Respond to the prompt with a thesis that analyzes the writer’s rhetorical choices.

• Select and use evidence to support your line of reasoning.

• Explain how the evidence supports your line of reasoning.

• Demonstrate an understanding of the rhetorical situation.

• Use appropriate grammar and punctuation in communicating your argument.

AP Lang Essay Example

Here is a decent if a bit shortened, AP Lang rhetorical analysis essay example you can use for reference.

Literature to Prepare for AP Lang

How to Write the AP Lang Synthesis Essay

And here is a list of some great AP Lang books that will help you prepare for the exam. Not all of them are immediately useful, but most will help you enhance your writing and analytical abilities to get a better score in the end.

  • The Odyssey
  • Don Quixote
  • A Midsummer Night's Dream
  • Pride and Prejudice
  • Wuthering Heights
  • Oliver Twist
  • Crime and Punishment
  • Adventures of Huckleberry Finn

If you have thoughts of "who could do my paper for me," do not forget that you can contact us. Or, if you have a finished paper and you need to make edits to it, leave us a ' rewrite my essay ' request and we will do it as soon as possible.

Daniel Parker

Daniel Parker

is a seasoned educational writer focusing on scholarship guidance, research papers, and various forms of academic essays including reflective and narrative essays. His expertise also extends to detailed case studies. A scholar with a background in English Literature and Education, Daniel’s work on EssayPro blog aims to support students in achieving academic excellence and securing scholarships. His hobbies include reading classic literature and participating in academic forums.

student sample ap lang synthesis essay

is an expert in nursing and healthcare, with a strong background in history, law, and literature. Holding advanced degrees in nursing and public health, his analytical approach and comprehensive knowledge help students navigate complex topics. On EssayPro blog, Adam provides insightful articles on everything from historical analysis to the intricacies of healthcare policies. In his downtime, he enjoys historical documentaries and volunteering at local clinics.

How to Write a Music Essay: Topics and Examples

  • Arizona State University (ASU)
  • Boston University
  • Dartmouth College
  • Georgetown University
  • Harvard University
  • Massachusetts Institute of Technology​
  • Our Contacts
  • Our Gallery
  • Our Pricing
  • Our Services
  • Purdue University Indianapolis (PU Indy)
  • Sample Page
  • universities
  • University of Florida
  • University of Michigan, Ann Arbor

How to Write the AP Lang Synthesis Essay with Example

How to Write the AP Lang Synthesis Essay with Example

Are you a high school student preparing for the AP Language and Composition exam? Or perhaps you are a teacher looking to help your students with the skills to ace the synthesis essay? Either way, you’ve landed in the right place. This blog will serve as your comprehensive guide to mastering this challenging yet rewarding component of the AP Language exam.

We’ll dig into what a synthesis essay entails and its structure, and we’ll furnish you with actionable strategies to approach the task with confidence. We’ll also provide insights into selecting and integrating sources effectively, constructing a compelling argument, and polishing your writing to perfection.

Table of Contents

Overview of AP Language and Composition

AP English Language and Composition , widely known as AP Lang, is a popular and engaging Advanced Placement course taken by over half a million high school students each year. The course is designed to hone essential skills such as analyzing written works, synthesizing information, constructing rhetorical essays, and writing compelling arguments. While the course presents a rigorous challenge, with just over 60% of students achieving a passing score of three or higher on the AP exam, the rewards of mastering these skills are significant.

The AP Lang exam is a comprehensive assessment consisting of two distinct sections. The first section, a one-hour multiple-choice segment, assesses your ability to analyze written passages and answer questions based solely on the provided text. This section comprises approximately 45% of the total exam score. The second section is a two-hour and fifteen-minute free-response segment. It evaluates your writing skills through three distinct essays. This section accounts for the remaining 55% of the exam score.

The three essays within the free-response section target specific writing skills. The synthesis essay challenges you to develop an argument by incorporating information from multiple provided sources. The rhetorical analysis essay requires you to dissect how an author uses language to convey meaning and achieve specific effects. Finally, the argumentative essay prompts you to take a stance on a debatable issue and construct a persuasive argument based on evidence.

What is the AP Lang Synthesis Essay?

The AP Language and Composition exam’s first free-response task is the synthesis essay. It is a one-hour exercise during which you read six to seven sources on a specific topic and compose a well-developed essay. These sources include a mix of print texts, approximately 500 words each, and visual elements like graphs or charts. You are advised to allocate 15 minutes to reading and analyzing these sources, followed by 40 minutes for writing and 5 minutes for review, but the time distribution can be adjusted as needed.

The synthesis essay prompt comprises three paragraphs: a brief introduction to the topic, a claim about the topic, and instructions for the essay. The claim is often broad and open to interpretation, requiring you to take a stance—either agreeing or disagreeing—and support your position by synthesizing information from at least three of the provided sources.

According to the College Board, a successful synthesis essay should “ combine different perspectives from sources to form a support of a coherent position. ” This means you must clearly state your claim, establish connections between sources to reinforce your argument, and provide specific evidence to validate your points.

The synthesis essay contributes six points to the overall AP Lang exam score. A holistic rubric evaluates the essay based on the thesis statement (0–1 point), evidence and commentary (0–4 points), and sophistication of thought and complexity of understanding (0–1 point).

Here’s an example prompt and essay provided by the College Board :


Urban rewilding is an effort to restore natural ecological processes and habitats in city environments. Many cities around the world have embraced rewilding as part of larger movements to promote ecological conservation and environmentally friendly design. Now, a movement to promote urban rewilding is beginning to take shape in the United States as well.



Refer to the sources as Source A, Source B, etc.; titles are included for your convenience.
Source A (infographic from Fastnacht)
Source B (Jepson and Schepers policy brief)
Source C (NRPA article)
Source D (Garland article)
Source E (graph from McDonald et al.)
Source F (Chatterton book excerpt)

In your response, you should do the following:
1. Respond to the prompt with a thesis that presents a defensible position.
2. Select and use evidence from at least three of the provided sources to support your line of reasoning.
3. Indicate clearly the sources used through direct quotation, paraphrase, or summary. Sources may be cited as Source A, Source B, etc., or by using the description in parentheses.
4. Explain how the evidence supports your line of reasoning.
5. Use appropriate grammar and punctuation when communicating your argument.

Rewilding is a term that not many people have heard of or even pay attention to. That doesn’t mean it’s not important, either. Rewilding is a good thing for the planet; it’s good for plants and the environment. The world needs to start caring, and children, especially, are the future.

Rewilding is good for our environment and for the future of preserving our world. In source C, “If people don’t spend any time outside, why are they going to care about their local places, let alone the national parks in the distance?” Going outside isn’t just good for the planet; it is also good for yourself. Nature isn’t really welcome in big cities, but reintroducing new plants can make it feel like it is welcome. Kids need to start caring about nature and not just about phones and video games. It gives you a different way to see our planet and care about what happens to it.

In addition, rewilding is valuable for our society to learn as a whole. In source B, “Rewilding is exciting, engaging, and challenging; it is promoting debate and deliberation on what is natural and the natures we collectively wish to conserve and shape.” It’s important for kids to understand, and a challenge can be what a lot of children need. Also in source A, “More than 70% of projected extinctions of plants and animals would be counteracted by restoring only 30% of priority areas.” That can be such a good thing, and that’s why rewilding, especially for our country, is important. If we don’t, we could lose 70% of plants and animals, which would send the ecosystem into whack.

Overall, rewilding should be focused on more; we have a lot to lose. Putting in the time and effort in our cities and urban settings is what we need to do. If you don’t care now, start caring. Kids especially need to focus.

Read also: Write an ap lang argument essay

How to Write a Synthesis Essay for the AP Language Exam

Step 1: analyze the prompt.

Begin by carefully reading and analyzing the prompt. Underline or highlight key terms to identify the central question and your task. Remember that you don’t need to decide your stance immediately; understanding the prompt is the priority here.

Step 2: Read and annotate the sources

Although you’ll only use three sources in your essay, read them all. This provides a broader understanding of the topic and helps you choose the most relevant evidence. As you read, actively annotate by highlighting key points, noting connections, and jotting down potential arguments.

After each source, briefly assess whether it supports, opposes, or nuances your emerging thesis. If you finish reading early, use the remaining time to start outlining your essay.

Step 3: Write a strong thesis statement

Your thesis statement should clearly state your position on the prompt’s claim. You can choose to defend the claim (argue it’s correct), challenge it (argue it’s incorrect), or qualify it (agree with some aspects and disagree with others). A strong thesis avoids summarizing the issue or restating the prompt; it establishes a clear line of reasoning.

Step 4: Outline your essay

Though it may seem counter intuitive when time is limited, outlining is essential. Your outline should include your thesis statement, three main points (one for each body paragraph), and the supporting evidence you’ll use from the sources. Briefly note how this evidence connects back to your thesis.

Step 5: Write your essay

With your annotated sources and outline in hand, writing your essay should be smoother. Begin with a focus on providing insightful commentary that explains how your evidence supports or refutes the prompt’s claim.

When referencing sources, use simple in-text citations like “Source 1,” “Source 2,” etc. Be sure to double-check your citations for accuracy. Before moving on, quickly proofread your essay for any errors.

Read also: How Long Should Your College Essay Be?

AP Lang Synthesis Essay Score Evaluation

The AP Language Synthesis Essay accounts for six points of the total exam score. Your essay will be evaluated on several key components. Primarily, a clear and defensible thesis statement that directly responds to the exam prompt can earn you up to one point. The majority of your score (up to four points) depends on how well you incorporate evidence from at least three sources and explain how that evidence supports your reasoning. Each piece of evidence should be explicitly linked to your argument, demonstrating a clear and consistent line of thought.

To earn the final point, your essay must show sophistication of thought. This can be achieved by writing a nuanced argument that acknowledges the complexities and tensions within the sources, situating your argument within a broader context to reveal its implications, or explaining the limitations of your or others’ arguments. Additionally, employing effective rhetorical devices and maintaining a vivid and persuasive writing style can further strengthen your essay.

Read also: Personal Statement Essay Examples

5 Tips to Ace the Synthesis Essay for the AP Language Exam

1. understand the prompt.

Begin by meticulously analyzing the prompt. Identify the central issue being discussed and the specific task you’re asked to perform (argue, evaluate, analyze, etc.). Underline key terms and phrases to ensure you fully grasp the expectations.

2. Engage actively with the sources

Don’t just skim through the sources; actively read and annotate the provided sources. Identify the main idea and supporting evidence in each. Note the source’s perspective and any potential biases. Highlight quotes or data you might use in your essay. Aim to understand how the sources relate to each other and the prompt.

3. Write a nuanced thesis

Your thesis should be a clear, concise statement of your position on the issue presented in the prompt. It should be specific and incorporate the nuances you’ve gleaned from the sources. Avoid merely restating the prompt; instead, offer an insightful perspective that you’ll support with evidence throughout your essay.

4. Construct a cohesive argument

Your essay should be a well-structured argument, not a mere summary of the sources. Each body paragraph should focus on a single point that supports your thesis. Use evidence from the sources to back up your claims, and provide your analysis and interpretation of that evidence. Connect your paragraphs with clear transitions to create a logical flow.

5. Leave time for revision

After writing your essay, take a few minutes to review it carefully. Check for grammatical errors, awkward phrasing, and clarity issues. Ensure that your argument is well-developed and your evidence is effectively integrated. A polished essay shows your command of language and strengthens your overall argument.

student sample ap lang synthesis essay

From the Desk of Yocket

Writing a good AP Language synthesis essay requires a balanced approach of critical thinking, careful analysis, and persuasive writing. You should begin by thoroughly understanding the prompt and identifying the central issue and the required task. Then, dig into the provided sources, extracting key points, perspectives, and evidence that relate to your developing stance.

A strong thesis is the backbone of your essay. It should clearly state your position on the issue and provide a roadmap for the reader, outlining the key points you’ll explore. As you bring together evidence from multiple sources, remember to provide insightful commentary, explaining how each piece of evidence bolsters your argument. Try to avoid simply dropping quotes or paraphrasing; instead, analyze the significance of each piece, showing a nuanced understanding of the issue and the sources.

You should conclude your essay by revisiting your thesis and summarizing your key arguments. You can also offer a thoughtful extension, such as suggesting implications for your argument, addressing potential counterarguments, or proposing future directions for research. Throughout your essay, prioritize clarity, coherence, and sophistication in your language and structure. This will show your ability to analyze complex texts and synthesize information into a compelling argument. Remember to maintain a strong connection with your audience, ensuring your writing on Yocket remains engaging and relevant.

Frequently Asked Questions

What is a synthesis essay on the ap language exam.

A synthesis essay requires you to develop a position on a given topic by incorporating and citing evidence from multiple sources. You’ll need to evaluate, select, and synthesize information from these sources to create a cohesive argument.

How many sources are typically provided for the synthesis essay?

The AP Language exam usually provides 6–7 sources for the synthesis essay, including texts and visual elements like graphs or charts.

What is the time allotted for writing the synthesis essay?

The entire free-response section of the AP Language exam, which includes the synthesis essay, rhetorical analysis, and argumentative essay, is 2 hours and 15 minutes. You may budget roughly 40 minutes to read the sources and plan your essay, leaving 40 minutes to write.

How is the synthesis essay scored?

The synthesis essay is scored on a 0–9 scale, with 9 being the highest. Points are awarded for a clear thesis, effective use of evidence and commentary, sophisticated analysis, and overall coherence.

Do I have to agree with the sources to use in my synthesis essay?

No, you can use sources to support a counterargument or provide alternative perspectives. The key is to engage with the sources critically and use them to build your argument.

How should I cite sources in my synthesis essay?

You can use parenthetical citations (author’s last name or source letter) to indicate where you’ve used information from the sources. It’s essential to avoid plagiarism by accurately attributing all borrowed ideas and language.

What are some common mistakes to avoid in the synthesis essay?

Try to avoid merely summarizing the sources without adding your analysis. Ensure your thesis clearly states your position and is supported by evidence throughout the essay. You should refrain from relying too heavily on one source and aim for a balanced incorporation of multiple perspectives.

  • No Comments
  • July 15, 2024

Leave a Reply Cancel reply

Your email address will not be published. Required fields are marked *

Save my name, email, and website in this browser for the next time I comment.

Are you seeking one-on-one college counseling and/or essay support? Limited spots are now available. Click here to learn more.

How to Write the AP Lang Argument Essay (With Example)

December 14, 2023

We’d like to let you in on a little secret: no one, including us, enjoys writing timed essays. But a little practice goes a long way. If you want to head into your AP English Exam with a cool head, you’ll want to know what you’re getting into ahead of time. We can’t promise the AP Lang Argument Essay will ever feel like an island vacation, but we do have tons of hand tips and tricks (plus a sample essay!) below to help you do your best. This article will cover: 1) What is the AP Lang Argumentative Essay? 2) AP Lang Argument Rubric 3) AP Lang Argument Sample Prompt 4) AP Lang Argument Essay Example 5) AP Lang Argument Essay Example: Answer Breakdown.

What is the AP Lang Argument Essay?

The AP Lang Argument Essay is one of three essays included in the written portion of the AP English Exam. The full AP English Exam is 3 hours and 15 minutes long, with the first 60 minutes dedicated to multiple-choice questions. Once you complete the multiple-choice section, you move on to three equally weighted essays that ask you to synthesize, analyze, and interpret texts and develop well-reasoned arguments. The three essays include:

Synthesis essay: You’ll review various pieces of evidence and then write an essay that synthesizes (aka combines and interprets) the evidence and presents a clear argument. Read our write-up on How to Write the AP Lang Synthesis Essay here.

Argumentative essay: You’ll take a stance on a specific topic and argue your case.

Rhetorical essay: You’ll read a provided passage, then analyze the author’s rhetorical choices and develop an argument that explains why the author made those rhetorical choices. Read our write-up on How to Write the AP Lang Rhetorical Essay here.

AP Lang Argument Essay Rubric

The AP Lang Argument Essay is graded on 3 rubric categories : Thesis, Evidence and Commentary, and Sophistication . How can you make sure you cover all three bases in your essay? We’ll break down each rubric category with dos and don’ts below:

  • Thesis (0-1 point)

When it comes to grading your thesis, AP Exam graders are checking off a box: you either have a clear thesis or you don’t. So, what crucial components of a thesis will get you your check mark?

  • Make sure your thesis argues something . To satisfy your graders, your thesis needs to take a clear stance on the issue at hand.
  • Include your thesis statement in your intro paragraph. The AP Lang Argumentative essay is just that: an essay that makes an argument, so make sure you present your argument right away at the end of your first paragraph.
  • A good test to see if you have a thesis that makes an argument for your AP Lang Argumentative Essay: In your head, add the phrase “I agree/disagree that…” to the beginning of your thesis. If what follows doesn’t logically flow after that phrase (aka if what follows isn’t an agreement or disagreement), it’s likely you’re not making an argument.
  • In your thesis, outline the evidence you’ll cover in your body paragraphs.

AP Lang Argument Essay Rubric (Continued)

  • Avoid a thesis that merely restates the prompt.
  • Avoid a thesis that summarizes the text but does not make an argument.
  • Avoid a thesis that weighs the pros and cons of an issue. Your job in your thesis is to pick a side and stick with it.
  • Evidence and Commentary (0-4 points)

This rubric category is graded on a scale of 0-4 where 4 is the highest grade. Unlike the rhetorical and synthesis essays, the evidence you need to write your AP Lang Argument Essay is not provided to you. Rather, you’ll need to generate your own evidence and comment upon it.

What counts as evidence?

Typically, the AP Lang Argument Essay prompt asks you to reflect on a broad cultural, moral, or social issue that is open to debate. For evidence, you won’t be asked to memorize and cite statistics or facts. Rather, you’ll want to bring in real-world examples of:

  • Historical events
  • Current-day events from the news
  • Personal anecdotes

For this essay, your graders know that you’re not able to do research to find the perfect evidence. What’s most important is that you find evidence that logically supports your argument.

What is commentary?

In this essay, it’s important to do more than just provide examples relevant evidence. After each piece of evidence you include, you’ll need to explain why it’s significant and how it connects to your main argument. The analysis you include after your evidence is commentary .

  • Take a minute to brainstorm evidence that logically supports your argument. If you have to go out of your way to find the connection, it’s better to think of different evidence.
  • Include multiple pieces of evidence. There is no magic number, but do make sure you incorporate more than a couple pieces of evidence that support your argument.
  • Make sure you include more than one example of evidence, too. Let’s say you’re working on an essay that argues that people are always stronger together than apart. You’ve already included an example from history: during the civil rights era, protestors staged group sit-ins as a powerful form of peaceful protest. That’s just one example, and it’s hard to make a credible argument with just one piece of evidence. To fix that issue, think of additional examples from history, current events, or personal experience that are not related to the civil rights era.
  • After you include each piece of evidence, explain why it’s significant and how it connects to your main argument.
  • Don’t summarize or speak generally about the topic. Everything you write must be backed up with specific and relevant evidence and examples.
  • Don’t let quotes speak for themselves. After every piece of evidence you include, make sure to explain and connect the evidence to your overarching argument.

AP Lang Argument Essay (Continued)

  • Sophistication (0-1 point)

According to the College Board , one point can be awarded to AP Lang Argument essays that achieve a high level of sophistication. You can accomplish that in four ways:

  • Crafting a nuanced argument by consistently identifying and exploring complexities or tensions.
  • Articulating the implications or limitations of an argument by situating it within a broader context.
  • Making effective rhetorical choices that consistently strengthen the force and impact of the student’s argument.
  • Employing a style that is consistently vivid and persuasive.

In sum, this means you can earn an additional point for going above and beyond in depth, complexity of thought, or by writing an especially persuasive, clear, and well-structured essay. In order to earn this point, you’ll first need to do a good job with the fundamentals: your thesis, evidence, and commentary. Then, to earn your sophistication point, follow these tips:

  • Outline your essay before you begin to ensure it flows in a clear and cohesive way.
  • Include well-rounded evidence. Don’t rely entirely on personal anecdotes, for example. Incorporate examples from current events or history, as well.
  • Thoroughly explain how each piece of evidence connects to your thesis in order to fully develop your argument.
  • Explore broader implications. If what you’re arguing is true, what does that mean to us today? Who is impacted by this issue? What real-world issues are relevant to this core issue?
  • Briefly explore the other side of the issue. Are the instances where your argument might not be true? Acknowledge the other side, then return to proving your original argument.
  • Steer clear of generalizations (avoid words like “always” and “everyone”).
  • Don’t choose an argument you can’t back up with relevant examples.
  • Avoid complex sentences and fancy vocabulary words unless you use them often. Long, clunky sentences with imprecisely used words are hard to follow.

AP Lang Argument Sample Prompt

The sample prompt below is published online by the College Board and is a real example from the 2021 AP English Exam. The prompt provides background context, essay instructions, and the text you need to analyze.

Suggested time—40 minutes.

Many people spend long hours trying to achieve perfection in their personal or professional lives. Similarly, people often demand perfection from others, creating expectations that may be challenging to live up to. In contrast, some people think perfection is not attainable or desirable.

Write an essay that argues your position on the value of striving for perfection.

In your response you should do the following:

  • Respond to the prompt with a thesis that presents a defensible position.
  • Provide evidence to support your line of reasoning.
  • Explain how the evidence supports your line of reasoning.
  • Use appropriate grammar and punctuation in communicating your argument.

AP Lang Argument Essay Example

As the old phrase says, “Practice makes perfect.” But is perfection something that is actually attainable? Sometimes, pushing for perfection helps us achieve great things, but most often, perfectionism puts too much pressure on us and prevents us from knowing when we have done the best we can. Striving for perfection can only lead us to shortchange ourselves. Instead, we should value learning, growth, and creativity and not worry whether we are first or fifth best.

Students often feel the need to be perfect in their classes, and this can cause students to struggle or stop making an effort in class. In elementary and middle school, for example, I was very nervous about public speaking. When I had to give a speech, my voice would shake, and I would turn very red. My teachers always told me “relax!” and I got Bs on Cs on my speeches. As a result, I put more pressure on myself to do well, spending extra time making my speeches perfect and rehearsing late at night at home. But this pressure only made me more nervous, and I started getting stomach aches before speaking in public.

Once I got to high school, however, I started doing YouTube make-up tutorials with a friend. We made videos just for fun, and laughed when we made mistakes or said something silly. Only then, when I wasn’t striving to be perfect, did I get more comfortable with public speaking.

AP Lang Argumentative Essay Example (Continued)

In the world of art and business and science, perfectionism can also limit what we are able to achieve. Artists, for example, have to take risks and leave room for creativity. If artists strive for perfection, then they won’t be willing to fail at new experiments and their work will be less innovative and interesting. In business and science, many products, like penicillin for example, were discovered by accident. If the scientist who discovered penicillin mold growing on his petri dishes had gotten angry at his mistake and thrown the dishes away, he would never have discovered a medicine that is vital to us today.

Some fields do need to value perfection. We wouldn’t like it, for example, if our surgeon wasn’t striving for perfection during our operation. However, for most of us, perfectionism can limit our potential for learning and growth. Instead of trying to be perfect, we should strive to learn, innovate, and do our personal best.

AP Lang Argument Essay Example: Answer Breakdown

The sample AP Lang Argumentative Essay above has some strengths and some weaknesses. Overall, we would give this essay a 3 or a 4. Let’s break down what’s working and what could be improved:

  • The essay offers a thesis that makes a clear argument that is relevant to the prompt: “Striving for perfection can only lead us to shortchange ourselves. Instead, we should value learning, growth, and creativity and not worry whether we are first or fifth best.”
  • The first body paragraph provides evidence that supports the essay’s thesis. This student’s personal anecdote offers an example of a time when perfectionism led them to shortchange themselves.
  • The second body paragraph provides additional evidence that supports the essay’s thesis. The example describing the discovery of penicillin offers another example of a situation in which perfectionism might have limited scientific progress.
  • The writer offers commentary explaining how her examples of public speaking and penicillin illustrate that we should “value learning, growth, and creativity” over perfectionism.
  • The essay follows one line of reasoning and does not stray into tangents.
  • The essay is organized well with intro, body, and concluding paragraphs. Overall, it is easy to read and is free of grammar errors.

What could be improved:

  • Although the second body paragraph provides one good specific example about the discovery of penicillin, the other examples it offers about art and business are only discussed generally and aren’t backed up with evidence. This paragraph would be stronger if it provided more examples. Or, if this writer couldn’t think of examples, they could have left out mentions of art and business altogether and included alternate evidence instead.
  • This writer would more thoroughly support their argument if they were able to offer one more example of evidence. They could provide another personal anecdote, an example from history, or an example from current events.
  • The writer briefly mentions the other side of the argument in their concluding paragraph: “Some fields do need to value perfection. We wouldn’t like it, for example, if our surgeon wasn’t striving for perfection during our operation.” Since it’s so brief a mention of the other side, it undermines the writer’s overall argument. This writer should either dedicate more time to reflecting on why even surgeons should “value learning, growth, and creativity” over perfectionism, or they should leave these sentences out.

AP Lang Argument Essay Example—More Resources

Looking for more tips to help you master your AP Lang Argumentative Essay? Brush up on 20 Rhetorical Devices High School Students Should Know and read our Tips for Improving Reading Comprehension .

If you’re ready to start studying for another part of the AP English Exam, find more expert tips in our How to Write the AP Lang Synthesis and How to Write the AP Lang Rhetorical Essay blog posts.

  • High School Success

Christina Wood

Christina Wood holds a BA in Literature & Writing from UC San Diego, an MFA in Creative Writing from Washington University in St. Louis, and is currently a Doctoral Candidate in English at the University of Georgia, where she teaches creative writing and first-year composition courses. Christina has published fiction and nonfiction in numerous publications, including The Paris Review , McSweeney’s , Granta , Virginia Quarterly Review , The Sewanee Review , Mississippi Review , and Puerto del Sol , among others. Her story “The Astronaut” won the 2018 Shirley Jackson Award for short fiction and received a “Distinguished Stories” mention in the 2019 Best American Short Stories anthology.

  • 2-Year Colleges
  • ADHD/LD/Autism/Executive Functioning
  • Application Strategies
  • Best Colleges by Major
  • Best Colleges by State
  • Big Picture
  • Career & Personality Assessment
  • College Essay
  • College Search/Knowledge
  • College Success
  • Costs & Financial Aid
  • Data Visualizations
  • Dental School Admissions
  • Extracurricular Activities
  • Graduate School Admissions
  • High Schools
  • Homeschool Resources
  • Law School Admissions
  • Medical School Admissions
  • Navigating the Admissions Process
  • Online Learning
  • Outdoor Adventure
  • Private High School Spotlight
  • Research Programs
  • Summer Program Spotlight
  • Summer Programs
  • Teacher Tools
  • Test Prep Provider Spotlight

“Innovative and invaluable…use this book as your college lifeline.”

— Lynn O'Shaughnessy

Nationally Recognized College Expert

College Planning in Your Inbox

Join our information-packed monthly newsletter.

IMAGES

  1. AP Lang Synthesis Essay Student Sample Activity by Jennifer Rodriguez

    student sample ap lang synthesis essay

  2. 😍 Ap language synthesis essay. Writing the Synthesis Essay. 2019-01-10

    student sample ap lang synthesis essay

  3. synthesis essay on ap lang exam

    student sample ap lang synthesis essay

  4. AP Lang Synthesis Essay

    student sample ap lang synthesis essay

  5. AP Lang Synthesis Essay Student Sample Activity by Jennifer Rodriguez

    student sample ap lang synthesis essay

  6. AP Lang Synthesis Essay Example: Prompt- Eminent Domain

    student sample ap lang synthesis essay

VIDEO

  1. Synthesis essay template

  2. Synthesis Activities

  3. What is AP Lang

  4. Synthesis Essay Conclusion Tips

  5. Do I Need to Read All the Synthesis Sources? #aplang

  6. Ab tbse sir uthe nahi hai 😂🤣#chetannn026 #funny #chetannn #comedy

COMMENTS

  1. Synthesis Essay Materials

    The two synthesis essay questions below are examples of the question type that has been one of the three free-response questions on the AP English Language and Composition Exam as of the May 2007 exam. The synthesis question asks students to synthesize information from a variety of sources to inform their own discussion of a topic. Students are given a 15-minute reading period to accommodate ...

  2. AP English Language and Composition Exam Questions

    Download free-response questions from this year's exam and past exams along with scoring guidelines, sample responses from exam takers, and scoring distributions. If you are using assistive technology and need help accessing these PDFs in another format, contact Services for Students with Disabilities at 212-713-8333 or by email at ssd@info ...

  3. PDF AP English Language and Composition Synthesis Essay Sample Student

    AP®English Language and Composition Synthesis Essay Sample Student Responses. The College Board: Connecting Students to College Success. The College Board is a not-for-profit membership association whose mission is to connect students to college success and opportunity. Founded in 1900, the association is composed of more than 5,000 schools ...

  4. PDF AP English Language and Composition

    In your response you should do the following: Respond to the prompt with a thesis that presents a defensible position. Select and use evidence from at least three of the provided sources to support your line of reasoning. Indicate clearly the sources used through direct quotation, paraphrase, or summary.

  5. How to Write the AP Lang Synthesis Essay + Example

    The synthesis essay counts for six points on the AP Lang exam. Students can receive 0-1 points for writing a thesis statement, 0-4 based on the incorporation of evidence and commentary, and 0-1 points based on the sophistication of thought and demonstration of complex understanding.

  6. How to Write the AP Lang Synthesis Essay with Example

    Understanding how these devices function can be essential in constructing a cohesive essay. Synthesis Essay AP Lang Examples - Sample Question . Below is a sample question from the AP Lang synthesis essay and a response to the prompt. This question was taken directly from a 2022 exam. However, the response to the question will be originally ...

  7. How to Write a Perfect Synthesis Essay for the AP Language Test

    While there are multiple essay examples online, we've chosen one to take a closer look at. We're going to give you a brief analysis of one of these example student synthesis essays from the 2019 AP Lang Exam below! Example Synthesis Essay AP Lang Response. To get started, let's look at the official prompt for the 2019 synthesis essay:

  8. PDF AP English Language and Composition Question 1: Synthesis (2019) Sample

    AP English Language and Composition Question 1: Synthesis (2019) Sample Student Responses 4 Sample A [1] According to a recent report on limate change published by the United Nations, Earth's global climate temperature is expected to rise by 1.5°C in the coming decades and the effects can potentially

  9. PDF ENGLISH LANGUAGE AND COMPOSITION

    AP English Language and Composition Question 1: Synthesis (2019) Sample Student Responses. 1. The student responses in this packet were selected from the 2019 Reading and have been rescored using the new rubrics for 2020. Commentaries for each sample are provided in a separate document.

  10. Acing the AP® English Language and Composition Synthesis Essay

    The newest section of the AP® English Language and Composition Exam, the synthesis essay, is one of three essays you will be completing during the examination's 2-hour free-response period. However, you'll also have a 15-minute reading and planning period just for this essay, and if you use this time to plan effectively, you can't go wrong.

  11. PDF AP English Language and Composition 2018 FRQ 1 Sample Student Responses

    AP English Language and Composition Question 1: Synthesis (2018) Sample Student Responses 3 Sample E [1] In the city of Baltimore, Maryland lie the district of Fells Point. This historically vibrant, lively corner of the sometimes-dull city is a refreshing retreat, yet during the late 20th

  12. How to Write the AP Lang Synthesis Essay + Essay Template

    The use of rhetorical devices AP Lang is also pretty important. So once you flesh out your essay a bit, spend some time trying to come up with the perfect wording. Step 5. Finalize. The first finished version of your essay is a draft. Don't be hasty to turn it in. Read over it a couple of times.

  13. PDF AP English Language and Composition

    AP ® English Language and Composition Sample Student Responses and Scoring Commentary Inside: ... Synthesis Essay 6 points . ... 2022 AP Exam Administration Student Samples and Commentary - AP English Language and Composition FRQ 1 Author: College Board

  14. How to Write the AP Lang Synthesis Essay with Example

    5 Tips to Ace the Synthesis Essay for the AP Language Exam. 1. Understand the prompt. Begin by meticulously analyzing the prompt. Identify the central issue being discussed and the specific task you're asked to perform (argue, evaluate, analyze, etc.).

  15. PDF ap06 english lang student samples

    The question directed students to read carefully an excerpt of William Hazlitt's 1827 essay, "On the Want of Money," and to analyze the rhetorical strategies the author uses to develop his position about money. Sample: 2A Score: 8. This essay's control and focus are evident from the first sentence.

  16. PDF AP Language Synthesis Essay Outline Example

    AP Language Synthesis Essay Outline Example The basic AP Language synthesis essay outline template contains three major parts: 1. Introduction with a thesis statement 2. Body, which contains arguments and counterarguments to the thesis 3. Conclusion Introduction An outline for a synthesis essay starts with an introduction, which is a brief ...

  17. PDF AP English Language and Composition

    AP ® English Language and Composition Sample Student Responses ... Synthesis Essay 6 points . ... The synthesis prompt for this year asked students to use material from six provided sources and develop a position on the place, if any, of handwriting instruction in today's schools. Students were expected to respond to

  18. How to Write the AP Lang Argument Essay (With Example)

    Typically, the AP Lang Argument Essay prompt asks you to reflect on a broad cultural, moral, or social issue that is open to debate. For evidence, you won't be asked to memorize and cite statistics or facts. Rather, you'll want to bring in real-world examples of: Historical events. Current-day events from the news.

  19. PDF AP English Language and Composition FRQ 1 Scoring Commentaries with

    AP English Language and Composition Question 1: Synthesis Scoring Commentaries on 2020 Rubrics (Applied to 2019 Student Responses) 3 September 2019 Sample A 6/6 Points (A1 - B4 - C1) Row A: 1/1 The response earned a point for Row A because it presents a clear thesis on the factors that should be

  20. AP Lang Synthesis Essay Example: Prompt- American Metaphors

    This essay is an example of a Synthesis Essay from AP English Language and Composition. This essay uses precise grammar and effective rhetorical devices to. Skip to document. University; High School. Books; Discovery. ... Students also viewed. Final draft copy - Bell Hooks Essay. Untitled document - To Kill A Mokingbird essay;

  21. PDF AP® ENGLISH LANGUAGE AND COMPOSITION

    The College Board. their. AP ENGLISH LANGUAGE AND COMPOSITION 2016 SCORING GUIDELINES. Question 1 (continued) 3- Essays earning a score of 3 meet the criteria for the score of 4 but demonstrate less success in arguing a position on whether monolingual English speakers are at a disadvantage today.

  22. PDF AP English Language and Composition Question 1: Synthesis 2020 Scoring

    AP English Language and Composition Question 1: Synthesis 2020 Scoring Commentaries (Applied to 2018 Student Responses) 2 September 2019 Sample I 6/6 Points (A1 - B4 - C1) Row A: 1/1 The response earned a point for Row A because it presents a thoughtful, multi-sentence thesis that indicates a clear position and establishes a line of reasoning.

  23. PDF AP® ENGLISH LANGUAGE AND COMPOSITION

    8 - Effective. Essays earning a score of 8 effectively argue a position on whether a school should establish, maintain, reconsider, or eliminate an honor code or honor system. They develop their argument by effectively synthesizing* at least three of the sources. The evidence and explanations used are appropriate and convincing.